Administración     

Olimpiadas de Matemáticas
Página de preparación y problemas

Selector
La base de datos contiene 1154 problemas y 775 soluciones.
OME Local
OME Nacional
OIM
OME Andalucía
Retos UJA
Problema 302
Se dan cinco números reales con la propiedad siguiente: independientemente de los tres que se elijan, la diferencia entre la suma de los elegidos y la suma de los restantes siempre es positiva. Probar que el producto de las diez diferencias no es mayor que el producto de los cuadrados de esos cinco números.
pistasolución 1info
Pista. Demostrar que la diferencia entre dos de los números es menor o igual que cualquiera de los otros tres números.
Solución. Llamemos $x_1\leq x_2\leq x_3\leq x_4\leq x_5$ a los cinco números dados ordenados de menor a mayor. Si $a,b,c,d,e$ son los números del $1$ al $5$ en algún orden, el enunciado nos dice que $$x_a+x_b+x_c\geq x_d+x_e,\qquad x_a+x_d+x_e\geq x_b+x_c.$$ Sumando estas desigualdades, obtenemos que $x_a\geq 0$ para todo $a$ entre $1$ y $5$. Si consideramos las desigualdades siguientes y las sumamos también: $$x_a+x_b+x_c\geq x_d+x_e,\qquad x_a+x_b+x_e\geq x_d+x_c,$$ obtenemos que $x_a-x_d\leq x_b$ para cualesquiera $a,b,d$ distintos entre $1$ y $5$. Con esto podemos acotar \begin{align*} x_5-x_4&\leq x_1,& x_5-x_3&\leq x_2,&x_5-x_2&\leq x_3,&x_5-x_1&\leq x_4,&x_4-x_3&\leq x_5,\\ x_4-x_2&\leq x_1,& x_4-x_1&\leq x_2,&x_3-x_2&\leq x_5,&x_3-x_1&\leq x_4,&x_2-x_1&\leq x_3.\\ \end{align*} Finalmente, si multiplicamos todas las desigualdades anteriores (todos sus términos son positivos), obtenemos el resultado buscado: $$\prod_{i\lt j}(x_i-x_j)\leq x_1^2x_2^2x_3^2x_4^2x_5^2.$$

Nota. Si ordenamos los números de mayor a menor $x_1\leq x_2\leq x_3\leq x_4\leq x_5$, la condición del enunciado se traduce simplemente en que $0\leq x_1$ y $x_5\leq x_1+x_2$. ¿Sabrías demostrarlo?

Si crees que el enunciado contiene un error o imprecisión o bien crees que la información sobre la procedencia del problema es incorrecta, puedes notificarlo usando los siguientes botones:
Informar de error en enunciado Informar de procedencia del problema
Problema 288
Dados números reales $x,y,z>0$ tales que $xyz=1$, demostrar que \[\frac{x^3}{(1+y)(1+z)}+\frac{y^3}{(1+z)(1+x)}+\frac{z^3}{(1+x)(1+y)}\geq\frac{3}{4}.\]
pistasolución 1info
Pista. La desigualdad de Chebyshev puede ser útil para transformar la desigualdad inicial.
Solución. Como la desigualdad es simétrica, podemos suponer que $x\geq y\geq z$ sin perder generalidad, de donde deducimos fácilmente que \[x^3\leq y^3\leq z^3,\qquad \frac{1}{(1+y)(1+z)}\leq \frac{1}{(1+x)(1+z)}\leq \frac{1}{(1+x)(1+y)}.\] De esta forma, podemos aplicar la desigualdad de Chebyshev, que nos asegura que \begin{eqnarray} \frac{x^3}{(1+y)(1+z)}+\frac{y^3}{(1+x)(1+z)}+\frac{z^3}{(1+x)(1+y)}&\geq&\frac{(x^3+y^3+z^3)\left(\frac{1}{(1+y)(1+z)}+\frac{1}{(1+x)(1+z)}+\frac{1}{(1+x)(1+y)}\right)}{3}\\ &=&\frac{(x^3+y^3+z^3)(3+x+y+z)}{3(1+x)(1+y)(1+z)} \end{eqnarray} Las desigualdades entre las medias aritmética y cúbica y entre las medias aritmética y geométrica nos permiten acotar \begin{eqnarray} x^3+y^3+z^3&\geq&\frac{1}{9}(x+y+z)^3,\\ 3(1+x)(1+y)(1+z)&\leq&\frac{1}{9}(3+x+y+z)^3 \end{eqnarray} Con estas dos desigualdades podemos transformar la desigualdad previa en \[\frac{x^3}{(1+y)(1+z)}+\frac{y^3}{(1+x)(1+z)}+\frac{z^3}{(1+x)(1+y)}\geq\frac{(x+y+z)^3}{(3+x+y+z)^2}.\] Ahora bien, la función $f(a)=\frac{a^3}{(3+a)^2}$ es estrictamente creciente para $a\geq 0$. Como $x+y+z\geq3\sqrt[3]{xyz}=3$, deducimos que $f(x+y+z)\geq f(3)=\frac{3}{4}$, de donde se deduce la desigualdad del enunciado.

Nota. Si la igualdad se alcanza, entonces del último razonamiento llegamos a que $x+y+z=3$, luego la igualdad en la desigualdad entre las medias nos asegura que $x=y=z=1$. Se comprueba que estos valores dan la igualdad y, por tanto, son los únicos.

Si crees que el enunciado contiene un error o imprecisión o bien crees que la información sobre la procedencia del problema es incorrecta, puedes notificarlo usando los siguientes botones:
Informar de error en enunciado Informar de procedencia del problema
Problema 285
Demostrar que si $x$, $y$ y $z$ son números reales positivos tales que $xyz=1$, entonces \[(1+x)(1+y)(1+z)\geq 8.\]
pistasolución 1info
Pista. La desigualdad entre las medias aritmética y geométrica puede ayudar.
Solución. Basta darse cuenta de que, por la desigualdad entre las medias aritmética y geométrica, se tiene que $1+x\geq2\sqrt x$, $1+y\geq2\sqrt y$ y $1+z\geq2\sqrt z$. Usando estas desigualdades, tenemos que \[(1+x)(1+y)(1+z)\geq 8\sqrt{xyz}=8.\]

Nota. La desigualdad $1+x\geq2\sqrt x$ es equivalente a $(1-\sqrt{x})^2\geq 0$, luego igualdad se alcanza cuando $x=y=z=1$.

Si crees que el enunciado contiene un error o imprecisión o bien crees que la información sobre la procedencia del problema es incorrecta, puedes notificarlo usando los siguientes botones:
Informar de error en enunciado Informar de procedencia del problema
Problema 281
Dados tres números reales positivos $a$, $b$ y $c$ tales que $a+b+c=1$, demostrar que \[\sqrt{a+bc}+\sqrt{b+ac}+\sqrt{c+ab}\leq 2{.}\]
pistasolución 1info
Pista. Aplica la desigualdad de Cauchy-Schwarz o la desigualdad entre las medias aritmética y cuadrática.
Solución. Aplicando la desigualdad entre las medias aritmética y cuadrática, obtenemos que \[\sqrt{a+bc}+\sqrt{b+ac}+\sqrt{c+ab}\leq 3\sqrt{\frac{a+b+c+ab+bc+ac}{3}}{.}\] Ahora bien, podemos usar que $a+b+c=1$ y que \[ab+bc+ac=\frac{(a+b+c) ^2-(a^2+b^2+c^2)}{2}=\frac{1-(a^2+b^2+c^2)}{2}\] para transformar la última expresión, obteniendo que \[\sqrt{a+bc}+\sqrt{b+ac}+\sqrt{c+ab}\leq 3\sqrt{\frac{3-(a^2+b^2+c^2)}{6}}{.}\] Usando de nuevo la desigualdad entre las medias aritmética y cuadrática deducimos que \[a^2+b^2+c^2\geq 3\left(\frac{a+b+c}{3}\right)^2=\frac{1}{3}{,}\] con lo que finalmente llegamos a que \[\sqrt{a+bc}+\sqrt{b+ac}+\sqrt{c+ab}\leq 3\sqrt{\frac{3-\frac{1}{3}}{6}}=2.\]

Nota. De la desigualdad entre las medias aritmética y cuadrática deducimos que la igualdad se alcanza si, y sólo si, $a=b=c=\frac{1}{3}$.

Si crees que el enunciado contiene un error o imprecisión o bien crees que la información sobre la procedencia del problema es incorrecta, puedes notificarlo usando los siguientes botones:
Informar de error en enunciado Informar de procedencia del problema
Problema 265
Sean $a$, $b$ y $c$ números reales positivos. Demostrar que \[a^ab^bc^c\geq(abc)^{(a+b+c)/3}{.}\]
pistasolución 1solución 2info
Pista. Toma logaritmos para eliminar los exponentes. Después hay varias posibilidades: una de ellas usando la desigualdad de Jensen y otra con la de reordenación.
Solución. Tomando logaritmos en la desigualdad del enunciado, ésta es equivalente a \[\frac{a\ln(a)+b\ln(b)+c\ln(c)}{a+b+c}\geq\frac{\ln(a)+\ln(b)+\ln(c)}{3}{.}\] Aplicando la desigualdad de Jensen a la función cóncava $f(x)=\ln(x)$ y a los números $x_1=a$, $x_2=b$ y $x_3=c$, con pesos $t_1=\frac{a}{a+b+c}$, $t_2=\frac{b}{a+b+c}$ y $t_3=\frac{c}{a+b+c}$, tenemos que \begin{eqnarray*} \frac{a\ln(a)+b\ln(b)+c\ln(c)}{a+b+c}&=&f(t_1x_1+t_2x_2+t_3x_3)\\ &\geq& t_1f(x_1)+t_2f(x_2)+t_3f(x_3)=\log\left(\frac{a^2+b^2+c^2}{a+b+c}\right). \end{eqnarray*} Por otro lado, las desigualdades entre las medias aritmética y cuadrática y entre las medias cuadrática y geométrica nos dicen que \[\sqrt{\frac{a^2+b^2+c^2}{3}}\geq\frac{a+b+c}{3},\qquad \sqrt{\frac{a^2+b^2+c^2}{3}}\geq\sqrt[3]{abc},\] y multiplicando estas dos desigualdades llegamos fácilmente a que \[\frac{a^2+b^2+c^2}{a+b+c}\geq\sqrt[3]{abc}{.}\] Usando esta desigualdad en el resultado que obtuvimos de la de Jensen, llegamos a que \[\frac{a\ln(a)+b\ln(b)+c\ln(c)}{a+b+c}\geq\log(\sqrt[3]{abc})=\frac{\ln(a)+\ln(b)+\ln(c)}{3}{,}\] que es la desigualdad buscada.

Nota. Otra forma de resolver este problema consiste en usar la desigualdad de Jensen sobre la función convexa $f(x)=x\ln(x)$.La igualdad se alcanza si, y sólo si, $a=b=c$, tal y como se deduce de la desigualdad de las medias o de la de Jensen.

Solución. Si suponemos que $a\leq b\leq c$, entonces $\ln(a)\leq\ln(b)\leq\ln(c)$ están ordenados en el mismo orden. Por lo tanto, tenemos que \begin{eqnarray*} a\ln(a)+b\ln(b)+c\ln(c)\geq b\ln(a)+c\ln(b)+a\ln(c),\\ a\ln(a)+b\ln(b)+c\ln(c)\geq c\ln(a)+a\ln(b)+b\ln(c),\\ a\ln(a)+b\ln(b)+c\ln(c) = a\ln(a)+b\ln(b)+c\ln(c). \end{eqnarray*} La primeras dos desigualdades se obtienen por la desigualdad de reordenación y la última es una igualdad trivial. Sumando las tres expresiones, llegamos a que \[a\ln(a)+b\ln(b)+c\ln(c)\geq\frac{a+b+c}{3}(\ln(a)+\ln(b)+\ln(c)),\] que es equivalente a la desigualdad propuesta, sin más que tomar logaritmos.
Si crees que el enunciado contiene un error o imprecisión o bien crees que la información sobre la procedencia del problema es incorrecta, puedes notificarlo usando los siguientes botones:
Informar de error en enunciado Informar de procedencia del problema
José Miguel Manzano © 2010-2024. Esta página ha sido creada mediante software libre